№ 10. Доказать неравенство
\((1) \quad \frac{1}{2}\cdot\frac{3}{4}...\frac{2n-1}{2n}<\frac{1}{\sqrt{2n+1}}\).

Решение. Прежде чем приступить к доказательству данного неравенства покажем следующее свойство действительных чисел.

Пусть \(0<a<1\), тогда \(\sqrt{a}<1\).

Действительно, если ввести обозначение \(\sqrt{a}=c\), тогда
\(c^2=a\),
\(c^2<1\),
\(c^2-1<0\),
\((*) \quad (c+1)(c-1)<1\).
Так как \(c>0\), поэтому \(c+1>0\) и неравенство (*) выполняется только в случае \(c-1<0\). Отсюда \(c<1\) и \(\sqrt{a}<1\).

Свойство доказано.

Для доказательства неравенства (1) воспользуемся методом математической индукции.

Покажем, что при \(n=1\) неравенство верно. Очевидно, что \(\sqrt{3}<2\). Разделив обе части этого неравенства на \(2\sqrt{3}\), получим:
\(\frac{1}{2}<\frac{1}{\sqrt{3}}\).
Неравенство (1) при \(n=1\) верно.

Теперь предположим, что неравенство (1) верно при \(n=k\):
\((2) \quad \frac{1}{2}\cdot\frac{3}{4}...\frac{2k-1}{2k}<\frac{1}{\sqrt{2k+1}}\).
и покажем, что оно также верно при \(n=k+1\). То есть необходимо показать верность неравенства
\(\frac{1}{2}\cdot\frac{3}{4}...\frac{2k-1}{2k}\cdot\frac{2(k+1)-1}{2(k+1)}<\frac{1}{\sqrt{2(k+1)+1}}\)
или же
\((3) \quad \frac{1}{2}\cdot\frac{3}{4}...\frac{2k-1}{2k}\cdot\frac{2k+1}{2k+2}<\frac{1}{\sqrt{2k+3}}\).

Из (2) следует, что
\(\frac{1}{2}\cdot\frac{3}{4}...\frac{2k-1}{2k}\cdot\frac{2k+1}{2k+2}<\frac{1}{\sqrt{2k+1}}\cdot\frac{2k+1}{2k+2}=\frac{\sqrt{2k+1}}{2k+2}\),
где
\(\frac{\sqrt{2k+1}}{2k+2}=\frac{1}{\sqrt{2k+3}}\cdot\frac{\sqrt{2k+3}\cdot\sqrt{2k+1}}{2k+2}=
\frac{1}{\sqrt{2k+3}}\cdot\sqrt{\frac{(2k+3)(2k+1)}{(2k+2)^2}}=\)
\(=\frac{1}{\sqrt{2k+3}}\cdot\sqrt{\frac{4k^2+8k+3}{4k^2+8k+4}}<\frac{1}{\sqrt{2k+3}}\).
Здесь последнее неравенство вытекает из доказанного выше свойства действительных чисел. Следовательно, неравенство (3) верно.

Неравенство (1) доказано.